Stay ahead of learning milestones! Enroll in a class over the summer!

Site Support Tech support and questions about AoPS classes and materials
Tech support and questions about AoPS classes and materials
3 M G
BBookmark  VNew Topic kLocked
Site Support Tech support and questions about AoPS classes and materials
Tech support and questions about AoPS classes and materials
3 M G
BBookmark  VNew Topic kLocked
G
Topic
First Poster
Last Poster
The order of colors
Entei   0
Apr 6, 2025
There are $3n$ balls, with $n$ red, $n$ green, and $n$ blue balls, randomly arranged in a row. Two observers, one at the front and one at the back, each record the order of the first appearance of each color. What is the probability that both observers record the same order of colors?

For example, the sequence RGGBRB would be read as RGB for the front observer and BRG for the back observer.
0 replies
Entei
Apr 6, 2025
0 replies
Problem in probability theory
Tip_pay   2
N Apr 4, 2025 by elizhang101412
Find the probability that if four numbers from $1$ to $100$ (inclusive) are selected randomly without repetitions, then either all of them will be odd, or all will be divisible by $3$, or all will be divisible by $5$
2 replies
Tip_pay
Apr 3, 2025
elizhang101412
Apr 4, 2025
Find the probability
ali3985   0
Apr 2, 2025
Let $A$ be a set of Natural numbers from $1$ to $N$.
Now choose $k$ ($k \geq 3$) distinct elements from this set.

What is the probability of these numbers to be an increasing geometric progression ?
0 replies
ali3985
Apr 2, 2025
0 replies
hard problem
Cobedangiu   1
N Apr 1, 2025 by Cobedangiu
Given any natural number $x$ satisfying $x \le k (k$ is a parameter; $k \in N$* and $x \vdots m(m \in N$*)
Write each natural number from $0 \to x$ on sheets of paper so that no sheet has the same number
Prove: The probability of drawing $x \in N$ satisfying of the given number is $\frac{[\frac{k}{m}] + 1}{k + 1}$
1 reply
Cobedangiu
Apr 1, 2025
Cobedangiu
Apr 1, 2025
Put this on a new Jane Street T-shirt
Assassino9931   0
Mar 30, 2025
Source: Bulgaria Spring Mathematical Competition 2025 12.3
Given integers \( m, n \geq 2 \), the points \( A_1, A_2, \dots, A_n \) are chosen independently and uniformly at random on a circle of circumference \( 1 \). That is, for each \( i = 1, \dots, n \), for any \( x \in (0,1) \), and for any arc \( \mathcal{C} \) of length \( x \) on the circle, we have $\mathbb{P}(A_i \in \mathcal{C}) = x$. What is the probability that there exists an arc of length \( \frac{1}{m} \) on the circle that contains all the points \( A_1, A_2, \dots, A_n \)?
0 replies
Assassino9931
Mar 30, 2025
0 replies
Student's domination
Entei   0
Mar 29, 2025
Given $n$ students and their test results on $k$ different subjects, we say that student $A$ dominates student $B$ if and only if $A$ outperforms $B$ on all subjects. Assume that no two of them have the same score on the same subject, find the probability that there exists a pair of domination in class.

Well, it seems like more clarity and formality are needed.

Given $n$ vectors in a $k$-dimensional space, $v_i = (x_{i1}, x_{i2}, \cdots, x_{ik})$, $i = 1, \cdots, n$. We say that a vector $v_i$ is dominant to another vector $v_j$ if and only if all components of $v_i$ are greater than those of $v_j$. Assume that no components are the same, that is $x_{ij} \neq x_{mn}, \forall i,j,m,n$.
Find the probability that there exists a pair of vectors that one is dominant to another.
0 replies
Entei
Mar 29, 2025
0 replies
two subsets with no fewer than four common elements.
micliva   37
N Mar 10, 2025 by Maximilian113
Source: All-Russian Olympiad 1996, Grade 9, First Day, Problem 4
In the Duma there are 1600 delegates, who have formed 16000 committees of 80 persons each. Prove that one can find two committees having no fewer than four common members.

A. Skopenkov
37 replies
micliva
Apr 18, 2013
Maximilian113
Mar 10, 2025
Did you talk to Noga Alon?
pohoatza   35
N Mar 2, 2025 by shendrew7
Source: IMO Shortlist 2006, Combinatorics 3, AIMO 2007, TST 6, P2
Let $ S$ be a finite set of points in the plane such that no three of them are on a line. For each convex polygon $ P$ whose vertices are in $ S$, let $ a(P)$ be the number of vertices of $ P$, and let $ b(P)$ be the number of points of $ S$ which are outside $ P$. A line segment, a point, and the empty set are considered as convex polygons of $ 2$, $ 1$, and $ 0$ vertices respectively. Prove that for every real number $ x$ \[\sum_{P}{x^{a(P)}(1 - x)^{b(P)}} = 1,\] where the sum is taken over all convex polygons with vertices in $ S$.

Alternative formulation:

Let $ M$ be a finite point set in the plane and no three points are collinear. A subset $ A$ of $ M$ will be called round if its elements is the set of vertices of a convex $ A -$gon $ V(A).$ For each round subset let $ r(A)$ be the number of points from $ M$ which are exterior from the convex $ A -$gon $ V(A).$ Subsets with $ 0,1$ and 2 elements are always round, its corresponding polygons are the empty set, a point or a segment, respectively (for which all other points that are not vertices of the polygon are exterior). For each round subset $ A$ of $ M$ construct the polynomial
\[ P_A(x) = x^{|A|}(1 - x)^{r(A)}.
\]
Show that the sum of polynomials for all round subsets is exactly the polynomial $ P(x) = 1.$

Proposed by Federico Ardila, Colombia
35 replies
pohoatza
Jun 28, 2007
shendrew7
Mar 2, 2025
Queer Dice
scls140511   2
N Feb 21, 2025 by SunnyEvan
Source: 2024 China Round 1 (Gao Lian)
5 Alice has an unfair dice such that the probability of rolling a $1$, $2$, $3$, $4$, $5$, $6$ forms an arithmetic sequence, in this order. Bob rolls the dice twice, getting $a$ and $b$ respectively. There is a $\frac{1}{7}$ probability that $a+b=7$. Find the probability such that $a=b$.
2 replies
scls140511
Sep 8, 2024
SunnyEvan
Feb 21, 2025
Covering 18 points with a figure
topologicalsort   1
N Feb 18, 2025 by Assassino9931
Source: Bulgarian Autumn Math Tournament 12.4
Let $L$ be a figure made of $3$ squares, a right isosceles triangle and a quarter circle (all unit sized) as shown below: IMAGE
Prove that any $18$ points in the plane can be covered with copies of $L$, which don't overlap (copies of $L$ may be rotated or flipped)
1 reply
topologicalsort
Nov 30, 2024
Assassino9931
Feb 18, 2025
a